LSAT Question Types Flashcards

1
Q

The pundit’s argument requires the assumption that

A

Necessary Assumption

How well did you know this?
1
Not at all
2
3
4
5
Perfectly
2
Q

Which one of the following is an assumption required by the argument?

A

Pseudo sufficient assumption

How well did you know this?
1
Not at all
2
3
4
5
Perfectly
3
Q

Which one of the following is an assumption on which the critic’s argument relies?

A

necessary assumption

How well did you know this?
1
Not at all
2
3
4
5
Perfectly
4
Q

Which one of the following most accurately describes how the scientist’s argument proceeds?

A

method of reasoning or descriptive

How well did you know this?
1
Not at all
2
3
4
5
Perfectly
5
Q

Which one of the following, if true, would most weaken the environment minister’s argument?

A

weaken

How well did you know this?
1
Not at all
2
3
4
5
Perfectly
6
Q

Which one of the following, if true, most helps to account for the response of auto safety experts to the popularity of SUVs?

A

resolve, reconcile or explain

How well did you know this?
1
Not at all
2
3
4
5
Perfectly
7
Q

The argument in the political advertisement is most vulnerable to criticism on the grounds that it

A

flaw or descriptive weakening

How well did you know this?
1
Not at all
2
3
4
5
Perfectly
8
Q

Which one of the following most accurately expresses the main conclusion of the client’s argument?

A

main conclusion or main point

How well did you know this?
1
Not at all
2
3
4
5
Perfectly
9
Q

Which one of the following, if true, most strengthens the support for the scientists’ hypothesis?

.

A

strengthen

How well did you know this?
1
Not at all
2
3
4
5
Perfectly
10
Q

Which one of the following principles, if valid, most strongly supports the reasoning in the argument?

A

strengthen

How well did you know this?
1
Not at all
2
3
4
5
Perfectly
11
Q

Which one of the following, if true, most calls into question the police chief’s explanation for the drop in crime?

A

weaken

How well did you know this?
1
Not at all
2
3
4
5
Perfectly
12
Q

The reasoning in the commentator’s argument is most vulnerable to criticism on the grounds that the argument

A

flaw or descriptive weakening

How well did you know this?
1
Not at all
2
3
4
5
Perfectly
13
Q

Which one of the following is most supported by Dr. Khan’s statements?

A

most strongly supported

How well did you know this?
1
Not at all
2
3
4
5
Perfectly
14
Q

Which one of the following most accurately describes the role played in the argument by the claim that society would not be better off if the government enacted laws requiring people to be polite to each other?

A

argument part

How well did you know this?
1
Not at all
2
3
4
5
Perfectly
15
Q

Which one of the following, if true, would most strengthen the astronomer’s argument?

A

strengthen

How well did you know this?
1
Not at all
2
3
4
5
Perfectly
16
Q

Which one of the following most accurately describes the role played in the argument by the claim that the greatest expense in irrigated agriculture is in pumping the water?

A

argument part

How well did you know this?
1
Not at all
2
3
4
5
Perfectly
17
Q

The statements above, if true, most strongly support which one of the following?

A

most strongly supported

How well did you know this?
1
Not at all
2
3
4
5
Perfectly
18
Q

The reasoning in the police captain’s argument is most vulnerable to criticism on the grounds that the argument

A

flaw or descriptive weakening

How well did you know this?
1
Not at all
2
3
4
5
Perfectly
19
Q

Which one of the following, if true of the economist’s country, most helps to resolve the apparent paradox in the economist’s statements?

A

resolve, reconcile or explain

How well did you know this?
1
Not at all
2
3
4
5
Perfectly
20
Q

Which one of the following conclusions is most strongly supported by the information above?

A

most strongly supported

How well did you know this?
1
Not at all
2
3
4
5
Perfectly
21
Q

Which one of the following, if true, does the most to support the prediction made by the device’s maker?

A

resolve, reconcile or explain

22
Q

Which one of the following is most appropriate as an analogy demonstrating that the reasoning in the argument above is flawed?

A

parallel flawed method of reasoning

23
Q

The editorial’s conclusion follows logically if which one of the following is assumed?

A

sufficient assumption

24
Q

The reasoning in the researcher’s argument is questionable in that the argument overlooks the possibility that

A

flaw or descriptive weakening

25
Q

The reasoning in which one of the following is most similar to the reasoning above?

A

parallel method of reasoning

26
Q

section 3 question 1

Which one of the following most logically completes the argument?

A

miscellaneous

27
Q

Which one of the following is an assumption required by the argument?

A

necessary assumption

28
Q

Which one of the following principles, if valid, most helps to justify the judgment that dismissing the complaint would be unfair?

A

pseudo sufficient assumption

29
Q

Which one of the following is most strongly supported by the information above?

A

most strongly supported

30
Q

Which one of the following, if true, helps to resolve the apparent conflict described above?

A

resolve reconcile or explain

31
Q

Which one of the following, if true, adds the most support for the hypothesis?

A

strengthen

32
Q

The reasoning in the employee’s argument is flawed because the argument

A

flaw or descriptive weakening

33
Q

Which one of the following most accurately expresses the overall conclusion drawn in the argument?

A

main conclusion or main point

34
Q

The pattern of reasoning in the argument above is most similar to that in which one of the following?

A

parallel method of reasoning

35
Q

The reasoning in the television host’s argument is flawed in that the argument

A

flaw or descriptive weakening

36
Q

The literature professor argues that the conclusion drawn by the critics has not really been established, on the grounds that

A

method of reasoning or descriptive

37
Q

Which one of the following, if true, justifies the above application of the policy?

A

sufficient assumption

38
Q

Which one of the following, if true, most seriously weakens the administrator’s argument?

A

weaken

39
Q

Which one of the following demonstrates most effectively by parallel reasoning that Branson’s argument is flawed?

A

parallel flawed method of reasoning

40
Q

The argument’s conclusion follows logically if which one of the following is assumed?

A

sufficient assumption

41
Q

The theorist’s argument is flawed in that it

A

flawed or descriptive weakening

42
Q

Which one of the following is an assumption the argument requires?

A

necessary assumption

43
Q

Which one of the following most accurately describes a reasoning flaw in the consumer advocate’s argument?

A

flaw or descriptive weakening

44
Q

Which one of the following is an assumption on which the argument depends?

A

necessary assumption

45
Q

Which one of the following most accurately describes the role played in the engineer’s argument by the statement that transportation will be much more expensive on semiplaning monohulls than on traditional ships?

A

argument part

46
Q

Which one of the following, if true, would most strengthen the argument?

A

strengthen

47
Q

Each of the following rejoinders, if true, directly counters the opponents’ explanation of the decrease in average family income EXCEPT:

A

except weaken

48
Q

The argument requires assuming which one of the following?

A

necessary assumption

49
Q

Which one of the following is most strongly supported by the statements made by the columnist?

A

most strongly supported

50
Q

The argument depends on assuming which one of the following?

A

necessary assumption